Difference between revisions of "2006 AIME I Problems/Problem 3"

m (See also)
m (box)
Line 1: Line 1:
 
== Problem ==
 
== Problem ==
Find the least positive integer such that when its leftmost digit is deleted, the resulting integer is 1/29 of the original integer.
+
Find the least [[positive]] [[integer]] such that when its leftmost [[digit]] is deleted, the resulting integer is <math>\frac{1}{29}</math> of the original integer.
  
 
== Solution ==
 
== Solution ==
Line 6: Line 6:
  
 
== See also ==
 
== See also ==
* [[2006 AIME I Problems/Problem 2 | Previous problem]]
 
* [[2006 AIME I Problems/Problem 4 | Next problem]]
 
* [[2006 AIME I Problems]]
 
 
* [[Number Theory]]
 
* [[Number Theory]]
 +
{{AIME box|year=2006|n=I|num-b=2|num-a=4}}
  
 
[[Category:Intermediate Number Theory Problems]]
 
[[Category:Intermediate Number Theory Problems]]

Revision as of 21:18, 11 March 2007

Problem

Find the least positive integer such that when its leftmost digit is deleted, the resulting integer is $\frac{1}{29}$ of the original integer.

Solution

The number can be represented as $10^na+b$, where $a$ is the leftmost digit, and $b$ is the rest of the number. We know that $b=\frac{10^na+b}{29} \implies 28b=2^2\times7b=10^na$. Thus $a$ has to be 7 since $10^n$ can not have 7 as a factor, and the smallest $10^n$ can be and have a factor of $2^2$ is $10^2=100.$ We find that $b$ is 25, so the number is 725.

See also

2006 AIME I (ProblemsAnswer KeyResources)
Preceded by
Problem 2
Followed by
Problem 4
1 2 3 4 5 6 7 8 9 10 11 12 13 14 15
All AIME Problems and Solutions